Roberto is x years older than his only sister but 10 years ago he was twice her age. What are the current ages of the siblings?

Answers

Answer 1

Answer:

S = x + 10

R = 2x + 10

Step-by-step explanation:

If R is Roberto's age, and S is his sister's age, then:

R = S + x

R − 10 = 2 (S − 10)

Solve with substitution.

S + x − 10 = 2 (S − 10)

S + x − 10 = 2S − 20

S = x + 10

R = 2x + 10


Related Questions

A pair of opposite vertices of a square is (1, 2) and (3,4). Find the coordinates of the remaining
vertices of the square.​

Answers

Answer:

(3, 2) and (1, 4)

Step-by-step explanation:

Plot the two points on a graph.

The other two points are (3, 2) and (1, 4).

To do this with algebra, it takes a few steps.

The diagonals of a square are perpendicular and bisect each other. You are given opposite vertices, so first, find the midpoint of that diagonal.

((1 + 3)/2, (2 + 4)/2) = (2, 3)

The midpoint of the diagonal is (2, 3).

This diagonal has slope 1 and y-intercept 1, so its equation is

y = x + 1

The perpendicular bisector has equation

y = -x + 5

The two vertices we are looking for, lie in a circle whose center is the midpoint of the diagonals, (2, 3), and whose radius is half of the diagonal.

Use Pythagoras to find the diagonal's length.

2^2 + 2^2 = c^2

c^2 = 8

c = sqrt(8) = 2sqrt(2)

Half of the diagonal is sqrt(2). This is the radius if the circle.

The equation of the circle is

(x - 2)^2 + (y - 3)^2 = (sqrt(2))^2

(x - 2)^2 + (y - 3)^2 = 2

The points of intersection of this circle and the second diagonal are the two vertices you are looking for.

System of equations:

(x - 2)^2 + (y - 3)^2 = 2

y = -x + 5

Use substitution and substitute y with -x + 5 in the equation of the circle.

(x - 2)^2 + (-x + 5 - 3)^2 = 2

(x - 2)^2 + (-x + 2)^2 - 2 = 0

x^2 - 4x + 4 + x^2 - 4x + 4 - 2= 0

2x^2 - 8x + 6 = 0

x^2 - 4x + 3 = 0

(x - 3)(x - 1) = 0

x - 3 = 0   or x - 1 = 0

x = 3 or x = 1

Now we find corresponding y values.

y = -x + 5

x = 3

y = -3 + 5 = 2

This gives us (3, 2).

y = -x + 5

x = 1

y = -1 + 5 = 4

This gives us (1, 4).

Answer: (1, 4) and (3, 2)

Which SMART goal attribute in the table applies to each of Devon’s financial goals?

Answers

Answer:

- specific goals: describe what exactly you want to achieve by doing something and how you will do that.

- measurable: which means you need to be able to identify the goals you want to achieve. You might have to break down your goal into measurable parts.

Step-by-step explanation:

Cory earns $9.50 per hour for the first 40 hours he works in a week. For any hours over 40 hours per week, his hourly rate is multiplied by 1.5. How much does he earn is he works for 43.5 hours in one week?

Answers

Answer:

429.88

Step-by-step explanation:

The first 40 hours is at 9.50

40 *9.50 =380

The remaining hours is at 9.50 * 1.5 or 14.25

43.5 -40 = 3.5

3.5 * 14.25

49.875 = 49.88  ( rounding to the nearest cent)

Add together

380+49.88 =429.88

Help please quick for c and d

Answers

Answer:

6.82 cm

Step-by-step explanation:

1000=V=pi*r^2*h. 1000=pi*r^3, r=6.82 cm

Please answer these 3 questions for 70 points 1 thanks 5 stars and brainiest

Answers

Answer:

Step-by-step explanation:

Problem 14: Mean 5.5, Range: 5.8

Problem 15: question #1 4, mode: 63

Probelm 16: Median 63, Range:  8.3

Answer:

14: Mean is 5.5 Hours

Range is 7.9 Hours

15: 4 Numbers

Mode is 6.3

16: Median is 6.3

Range is 8.3

Step-by-step explanation:

(I'll edit in the explanations momentarily)

Hey please help me with Math!
There’s a bicycle that runs 45km in 1 hour and 15 minutes.
1) What is the speed of this bicycle? (__km/ph)
2) How far will this bicycle get in 2 hours and 40 minutes?

Answers

Step-by-step explanation:

Hello!!!!

According to the question,

distance covered (d)= 45km.

time taken (t)= 1 hr and 15 mins.

now,

let's find answer of this question no. i).

answer :

speed (s)= distance by time taken

= 45 /1.25

so, the speed is 36 km/hr.

now, for 2nd question,

time (t)= 2hrs and 40 mins.

speed (s)= 36km / hr

now,

distance (d)= s×t

= 36×2.6666666667

= 96km....is yourrequired answer.

Hope it helps...

a polynomial has been factored below but some constants are missing. 2x^3-8x^2-24x=ax(x+b)(x+c)

Answers

Answer:

The polynomial is 2x^3 - 8x^2 - 24x

And we can factor out a 2x from each of the three terms:

2x(x^2 - 4x - 12)

Lastly, factor the remaining quadratic:

2x(x+(-2))(x+6)

And we have our answer:

a=2

b=-2

c=6

Let me know if this helps!

Answer:

a =2, b =2, and c = -6

Step-by-step explanation:

We factor the polynomial and then see which value corresponds to what.

2x^3-8x^2-24x

As we see it, all terms are factorable by 2x. So if we take out 2x from every term, we get

2x(x^2 - 4x - 12)

Now we factor the quadratic, which we can do mentally to get

2x(x+2)(x-6)

ax(x+b)(x+c)

Comparing that to ax(x+b)(x+c), we can tell that a =2, b =2, and c = -6.

The mean amount purchased by a typical customer at Churchill's Grocery Store is $23.50 with a standard deviation of $5.00. Assume the distribution of amounts purchased follows the normal distribution. For a sample of 50 customers, answer the following questions.
(a) What is the likelihood the sample mean is at least $25.00? (Round z value to 2 decimal places and final answer to 4 decimal places.)
Probability
(b) What is the likelihood the sample mean is greater than $22.50 but less than $25.00? (Round z value to 2 decimal places and final answer to 4 decimal places.)
Probability
(c) Within what limits will 90 percent of the sample means occur? (Round your answers to 2 decimal places.)

Answers

Answer:

a. [tex]\mathtt{P(X \geq 25) =0.0170}[/tex]     ( to four decimal places)

b. [tex]P(22.5<X<25) = 0.9043[/tex]   ( to four decimal places )

c. The limits will be between the interval of   ( 22.33,24.67 )

Step-by-step explanation:

Given that :

mean = 23.50

standard deviation = 5.00

sample size = 50

The objective is to calculate the following:

(a)  What is the likelihood the sample mean is at least $25.00?

Let X be the random variable, the probability that the sample mean is at least 25.00 is:

[tex]P(X \geq 25) = 1 - P(\dfrac{X - \mu}{\dfrac{\sigma}{\sqrt{n}}} < \dfrac{25- 23.50}{ \dfrac{5}{\sqrt{ 50}} })[/tex]

[tex]P(X \geq 25) = 1 - P(Z< \dfrac{1.5}{ \dfrac{5}{7.07107}} })[/tex]

[tex]P(X \geq 25) = 1 - P(Z< \dfrac{1.5 \times 7.071}{ {5}})[/tex]

[tex]P(X \geq 25) = 1 - P(Z< 2.1213)[/tex]

[tex]P(X \geq 25) = 1 - P(Z< 2.12)[/tex]   to two decimal places

From the normal tables :

[tex]P(X \geq 25) = 1 - 0.9830[/tex]

[tex]\mathtt{P(X \geq 25) =0.0170}[/tex]     ( to four decimal places)

(b) What is the likelihood the sample mean is greater than $22.50 but less than $25.00?

[tex]P(22.5<X<25) = P(\dfrac{X-\mu}{\dfrac{\sigma}{\sqrt{n}}} <\dfrac{25-23.5}{\dfrac{5}{\sqrt{50}}} ) - P(\dfrac{X-\mu}{\dfrac{\sigma}{\sqrt{n}}} <\dfrac{22.5-23.5}{\dfrac{5}{\sqrt{50}}} )[/tex]

[tex]P(22.5<X<25) = P(Z<\dfrac{1.5}{\dfrac{5}{7.071}} ) - P(Z<\dfrac{-1}{\dfrac{5}{7.071}} )[/tex]

[tex]P(22.5<X<25) = P(Z<2.12) - (Z<-1.41 )[/tex]

[tex]P(22.5<X<25) = (0.9830 ) - (0.0787)[/tex]

[tex]P(22.5<X<25) = 0.9043[/tex]  to four decimal places

(c) Within what limits will 90 percent of the sample means occur?

At 90 % confidence interval, level of significance = 1 - 0.90 = 0.10

The critical value for the [tex]z_{\alpha/2} = 0.05[/tex] = 1.65

Standard Error = [tex]\dfrac{\sigma}{\sqrt{n}}[/tex]

Standard Error =  [tex]\dfrac{5}{\sqrt{50}}[/tex]

Standard Error = 0.7071

Therefore, at 90 percent of the sample means, the limits will be between the intervals of : [tex](\mu \pm z_{\alpha/2} \times S.E)[/tex]

Lower limit =  ( 23.5 - (1.65×0.707) )

Lower limit =  ( 23.5 - 1.16655 )

Lower limit = 22.33345

Lower limit = 22.33    (to two decimal places).

Upper Limit = ( 23.5 + (1.65*0.707) )

Upper Limit = ( 23.5 + 1.16655 )

Upper Limit = 24.66655

Upper Limit = 24.67

The limits will be between the interval of   ( 22.33,24.67 )

(x^2-4x)^2+7x^2-28x+12=0

Answers

Answer:

[tex]x^4-9x^2-28x=-12[/tex]

Step-by-step explanation:

[tex](x^2-4x)^2+7x^2-28x+12=0[/tex]

[tex](x^4-16x^2)+7x^2-28x=-12[/tex]

[tex]x^4-9x^2-28x=-12[/tex]

fill in the table with whole numbers to make 2.8 in three different ways i do not get this qestion can you help me

Answers

Answer: what table? u need to add an attachment

Step-by-step explanation:

The table containing whole number express 2.8 as the sum or difference.

To express 2.8 as the sum or difference of whole numbers.

There are three different ways to do it:

1. As a sum of whole numbers:

  2 + 0.8 = 2.8

2. As a difference of whole numbers:

  4 - 1.2 = 2.8

3. Another way as a sum of whole numbers:

  1 + 1.8 = 2.8

The table will be:

Representation             Whole Number 1        Whole Number 2      Result

          1                                      2                               0.8                       2.8

          2                                      4                               1.2                       2.8

          3                                      1                               1.8                        2.8

As each row represents a different way to represent the number 2.8 using whole numbers.

Learn more about Whole number here:

https://brainly.com/question/29766862

#SPJ3

Draw the image of △ABC under a dilation whose center is P and scale factor is 1/2.

Answers

Answer:

  See below

Step-by-step explanation:

Each point on the triangle moves to half its previous distance from P.

Please answer this question now

Answers

Answer:

95 degrees

Step-by-step explanation:

Measure of arc AB is 360 - (101+97+69) = 360 - 267 = 93 degrees.

Measure of arc DAB is 97+93 = 190 degrees, so measure of angle C is 190/2 =  95 degrees.


HELP! SUPER URGENT!

A water sports store hires out wetsuits and diving equipment. The cost to hire a wetsuit is one-third of the cost to hire diving equipment.

Dave hired both a wetsuit and diving equipment for the same length of time and paid a total of $144.

How much did he pay for the hire of the wetsuit in dollars?

Answers

Answer:

Let the cost of hiring diving equipment be X

Given,

cost of hiring diving equipment=X

cost of hiring wet suit =1/3 of X

Total cost of both =$144

Now,

X+1/3X=$144

or, (3X+X)/3=$144 ( taking l.c.m.)

or, 4X/3=$144

or, 4X=$144*3

or, X=$432/4

or, X=$108

Therefore, he paid $36 which is 1/3 of x to hire wetsuit.

Please help, 50 points! :)
PDF attached below

Answers

Answer:

See below

Step-by-step explanation:

Attachment 1 : (a) Remember that it mentions x is the years since 1900. That would mean that the table is a bit different. To create this " new table " simply subtract 1900 from the years provided, and substitute.

To create this equation we will need a regression calculator. The equation will be as follows.

y = 0.125873x - 7.11916 ( note that you can double check this equation be substituting points from the table in the attachment )

(b) 2025 - 1900 = 126 years,

y = 0.125873(125) - 7.11916 = $ 8.614965

Minimum Wage : $ 8.614965

Attachment 2 : The rest of the problems can be solved similarly...

(a) Quadratic Regression Equation : - 0.49311x² + 23.2798x + 996.029

(b) - 0.49311(20)² + 23.2798(20) + 996.029 = 1264.381 mg/cm³

Attachment 3 : (a) Exponential Regression : 9.08292(1.09965)ˣ

(b) 9.08292(1.09965)⁶⁰ = [tex]2713.27743\dots[/tex] ( About 2713 recommendations )

Work out an estimate for the value of 89.3X0.51divided by 4.8​

Answers

Answer:

It is 9.5

Step-by-step explanation:

[tex] = (89.3 \times 0.51 )\div 4.8[/tex]

first solve the bracket:

[tex] = (45.543) \div 4.8 \\ = 9.488[/tex]

PLEASE HELP ME I WILL GIVE 5 STARS TO THE FIRST ONE WHO GETS THIS RIGHT !

Answers

Answer:

Option 1 and option 4

Step-by-step explanation:

The inverse of 12^2 is the √12 so option 4 is correct. 12^1/2 also equals √12 so option 1 is also correct.

Answer:

12 [tex]\frac{2}{1}[/tex]

Step-by-step explanation:

pls help me asap !!!!!

Answers

Answer:

9

Step-by-step explanation:

This is a right triangle.  We can use the Pythagorean theorem to determine the missing side.

a^2 + b^2 = c^2 where a and b are the legs and c is the hypotenuse

a^2 + 12 ^2 = 15^2

a^2 +144 =225

a^2 = 225-144

a^2 = 81

Taking the square root of each side

a = 9

Answer:

9

Step-by-step explanation:

Because the triangle is a right triangle we can use the Pythagorean Theorem to solve for the missing side.

Pythagorean Theorem: a² + b² = c²

where

a = short leg

b = long leg

c = hypotenuse ( longest side )

we are given the hypotenuse and long leg and need to find the short leg

given hypotenuse (c) = 15

given long leg (b) = 12

short leg (a) = ?

to find the short leg plug in the known values into the Pythagorean Theorem and solve for a

a² + b² = c²

b = 12 and c = 15

a² + 12² = 15²

solve for a

a² + 12² = 15²

apply exponents

a² + 144 = 225

subtract 144 from both sides

a² = 81

take the square root of both sides

√a² = √81

a = 9

What is the measure of

Answers

Answer:

60⁰ is the answer. akksjdjd

Please help me solve this question!

Answers

I hope you get right answer.

write as a numeral in simplest form:(6x1000)+(4x100)+(6x1)

Answers

answer:
6401
step-by-step explanation:
6 x 1000
6000
4 x 100
400
6 x 1
6
6401

find square root of: 91+9, 47+2, 19+125, 9+0

Answers

Answer:

√(91+9)

=√100

= 10

√(47+2)

=√49

= 7

√(19+125)

=√144

= 12

√(9+0)

=√9

=3

10, 7, 12 and 3 is the square root to all of those :)

Choose the composition of translations that maps the image ΔPQR onto ΔP′Q′R′.

Question 4 options:

(x,y) → (x + 6,y + 3)


(x,y) → (x – 6,y – 3)


(x,y) → (x + 6,y – 3)


(x,y) → (x – 6,y + 3)

Answers

Answer:

(x , y )  ---> (x + 6 , y - 3)

Step-by-step explanation:

(x , y )  ---> (x + 6 , y - 3)

P(-1 , 4) -----> P'(-1+6 , 4-3) = P'(5,1)

Comapre P and P' x-coordinate

         -1 + a = 5

                a = 5 +1 = 6

Q(-1, 2) ----->Q'(-1+6 , 2 -3)= Q'(5, -1)

R(3 , 1) ------> R'(3+6 , 1-3)  = R'(9,-2)

the product of the product of 7 and 2 and the product of 8 and p

Answers

Answer:

((7*2)*(8p))

Step-by-step explanation:

The product of 7 and 2 basically means you multiply them. The product of 7 and 2 is 14 and the product of 8p cannot be known because p is undefined so keep it as 8p. the product of 14 and 8p is 112p but the answer I stated above is the unsimplified version.

Geraldo recently saw a newspaper ad for a new version of his laptop. The projected price is $400.00, and the laptop will be out on the market in about one year. Geraldo wants to purchase a new laptop but is wondering if he should wait a year. With 2.5% inflation, what amount would he pay to purchase a laptop today that is the same value as the one he saw in the ad?

Answers

Answer:

The answer would be $410.00.

Cheers,

Got an question worth 25 points, pls guys go and answer it. Find the question on my profile.

Thankssss.

The price he will have to pay now for laptop will be $390.

What is cost prize ?

The prize at which the good and services have been bought is known as cost prize.

here, the given information is :

The projected price is $400.00, and the laptop will be out on the market in about one year with 2.5% inflation.

Now, if he pay to purchase a laptop today that is the same value as the one he saw in the ad then the cost price he will have to pay will be 2.5% of $400 less that is:

cost price of laptop = $400 - 2.5& x 400

cost price of laptop = 97.5% x $400

cost price of laptop = $390

Therefore, the price he will have to pay now for laptop will be $390.

check and know more about cost price here :

https://brainly.com/question/11027396

#SPJ2

need help pls will give you 5 stars

Answers

Answer:

f(x + k)

Step-by-step explanation:

A horizontal shift moves to the left or right

y = f(x + k)  k > 0 moves it left

                   k < 0 moves it right

Find the number of terms of the geometric progression 81, 27, 9,....,1/81

Answers

Answer:

9

Step-by-step explanation:

r = 27/81 = 1/3

a = 81

Tn = 1/81

ar^(n-1) = 1/81

81(1/3)^(n-1) = 1/81

(1/3)^(n-1) = 1/6561

(1/3)^(n-1) = 1/3^(8)

Compare the exponents,

n-1 = 8

n = 9

2. If Bill has an apple, an orange, a pear, a grapefruit, a banana, and a kiwi at home and he wants to bring three pieces of fruit to school, how many combinations of fruit can he bring?

Answers

Answer:

6 choose 3 = 20

Step-by-step explanation:

[tex]=\frac{n!}{r!\left(n-r\right)!}[/tex]

6 choose 3 = 20

The total combinations of fruit he can bring is 20 if Bill has an apple, an orange, a pear, a grapefruit, a banana, and a kiwi at home and he wants to bring three pieces of fruit to school the answer is 20.

What are permutation and combination?

A permutation is the number of different ways a set can be organized; order matters in permutations, but not in combinations.

We have:

Bill has an apple, an orange, a pear, a grapefruit, a banana, and a kiwi at home and he wants to bring three pieces of fruit to school

Total number of fruits = 1+1+1+1+1+1

Total number of fruits = 6

n = 6

He wants to bring three pieces of fruit to school,

r = 3

Total combination:

= C(6, 3)

= 6!/(6-3)!3!

= 720/(6)(6)

= 720/36

= 20 combination

Thus, the total combinations of fruit he can bring is 20 if Bill has an apple, an orange, a pear, a grapefruit, a banana, and a kiwi at home and he wants to bring three pieces of fruit to school the answer is 20.

Learn more about permutation and combination here:

https://brainly.com/question/2295036

#SPJ5

Mia’s house and her aunt’s house are 15.4 inches apart on the map. If every 4 inches on the map represents 10 miles, what is the actual distance from Mia’s house to her aunt’s house, to the nearest tenth of a mile? 2.6 6.2 38.5 61.6

Answers

Answer:

38.5 miles

Step-by-step explanation:

Proportions:

4 inches ⇔ 10 miles

15.4 inches ⇔ M miles

M = 15.4*10/4

M = 38.5 miles

Answer:  38.5    

Step-by-step explanation: cuz im smart

Kelly bought a crate of floor tiles for $95.94. The crate had 6 boxes of floor tiles. Each box contained 20 floor tiles.

Write and solve an equation to determine the cost per box, b. Then write and solve a second equation to determine the cost per tile, t, to the nearest cent.

Answers

Answer:

$1.60 a crate

Step-by-step explanation:

t= 95.94/(6x20)

(6x20)= 60

95.94/60

$1.60

Answer:

Step-by-step explanation:

i) Cost per box = cost of a crate ÷ Number of boxes in the crate

b = 95.94 ÷ 6

b = $ 15.99

ii) Cost per tile = Cost per box ÷ Number of tiles in a box

 t = b ÷ 20

 t = 15.99 ÷20

t = $ 0.7995

Find: 499 decreased by 100%

Answers

Answer:

0,

If you want, I can explain more into it.

Other Questions
How do u simplify each expression by combining like terms? Mr. Howe ate 1/3 of a pizza and then Mr. Kurt ate 1/8 of the same pizza. Howmuch of the pizza has been eaten? * Write an expression to represent the given statement. Use n for the variable. Three times the absolute value of the sum of a number and 6 At the "cloth for you" shop, you can buy a top for 10.00 and a Bermuda trouser for 12.00. Due to a sensational sell, there is a 20% discount on all tops. If you buy one top and two Bermuda trousers, how much money do you spend in total? If 1 mol of a pure triglyceride is hydrolyzed to give 2 mol of RCOOH, 1 mol of R'COOH, and 1 mol of glycerol, which of the following compounds might be the triglyceride? CHOC(O)R A. CHOC(O)R CHOC(O)R CH,OC(O)R B. CHOC(O)R CH2OC(O)R CHOC(O)R C. CHOC(O)R CHOC(O)R CHOC(O)R D. CHOC(O)R CHOC(O)R In Act 1, scene 5 from Shakespeares Twelfth Night, Feste says to Olivia, "I wear not motley in my brain." What does he mean? joe went to to the store and spent a total of $37 .84. if paid with a $50 then how much change did he get back? NEED ANSWER FAST....FIRST ANSWER WILL BE BRAINLIST!!!!For which data set would the mean absolute deviation be a good indicator of variation of the data? Find the graph of the inequality y Which of the following is an example of the geosphere interacting with the atmosphere?Streams become muddy due to erosion. Volcanic eruptions release gases into the air. Rocks crack open when water in the cracks freeze. Plants absorb carbon dioxide and produce oxygen. D(x2 + y2 + 2020)dxdy D: x2 +y2 +2ax 0 (a > 0) GUYS PLEASE HELP WILL MARK BRAINLIEST A plastic rod that has been charged to 15 nC touches a metal sphere. Afterward, the rod's charge is 5.0 nC.1) What kind of charged particle was transferred between the rod and the sphere, and in which direction? A) electrons transferred from rod to sphere. B) electrons transferred from sphere to rod. C) protons transferred from rod to sphere. D) protons transferred from sphere to rod.2) How many charged particles were transferred? SHOW ADEQUATE WORKINGS IN THIS SECTION12. Wale and Lekan of 55kg and 60kg respectively ran a race of 200m. i. Calculate their work done in KJii. If wale finished the race in 25secs while Lekan finished in 30secs, calculate their power and who is more powerful out of these two. ( g = 10m/secs)13. A machine has a velocity ratio of 5 and is 800/0 efficient. If the machine is carrying a load of 200kg, what will be effort applied?Please help me answer anyone that you understand Can someone help i need this done by today Using Article 26 from the Universal Declaration of Human Rights, summarize then analyze the article in light of what you know of the education in other countries that do not uphold this right. Include the different sections and discuss the effects on a country if every country involved implemented this right.NEED ASAP!! Article 26 below (1) Everyone has the right to education. Education shall be free, at least in the elementary and fundamental stages. Elementary education shall be compulsory. Technical and professional education shall be made generally available and higher education shall be equally accessible to all on the basis of merit.(2) Education shall be directed to the full development of the human personality and to the strengthening of respect for human rights and fundamental freedoms. It shall promote understanding, tolerance and friendship among all nations, racial or religious groups, and shall further the activities of the United Nations for the maintenance of peace. (3) Parents have a prior right to choose the kind of education that shall be given to their children. What is the slope and the y-intercept of the line on the graph below? On a coordinate plane, a line goes through points (0, 1) and (4, 0). Simplify: [tex] \sqrt[4]{6 ^{4} } [/tex] Practice Exercise 3.1 Fill in the blanks: (i) The factors of 12 are (ii) The least non-zero multiples of any number is (iii) ......... is a factor of every number. ing with Numbers Digby's balance sheet has $99,131,000 in equity. Further, the company is expecting net income of 3,000,000 next year, and also expecting to issue $4,000,000 in new stock. If there are no dividends paid what will beDigby's book value